Get Answers to all your Questions

header-bg qa

A and B are events such that P(A\cup B)=3/4, P(A\cap B)=1/4, P(\bar{A})=2/3\; \; then\; \; P(\bar{A}\cap B)  is

  • Option 1)

    5/12

  • Option 2)

    3/8

  • Option 3)

    5/8

  • Option 4)

    1/4     

 

Answers (1)

best_answer

As we learnt in 

Algebra of events -

[ If A and B are mutually exclusive events then A\cap B= \phi

\therefore P\left ( A\cap B \right )= 0

\therefore P\left ( A\cup B \right )= P\left ( A \right )+P\left ( B \right )

\Rightarrow P\left ( A\cup B\cup C \right )= P\left ( A \right )+\left P( B \right )+P\left ( C \right )

\Rightarrow P\left ( \overline{A}\cap \overline{B} \right )= 1-P\left ( A\cup B \right )

\Rightarrow P\left ( \overline{A}\cup \overline{B} \right )= 1-P\left ( A\cap B \right )

\Rightarrow P \left ( A \right )=P\left ( A\cap B \right )+P\left ( A\cap \overline{B} \right )

\Rightarrow P \left ( B\right )=P\left ( B\cap A \right )+P\left ( B\cap \overline{A} \right ) ]

-

 P(A\cup B)=P(A)+P(B)-P(A\cap B)

\frac{3}{4}=\frac{1}{3}+(P(B)-\frac{1}{4}

\Rightarrow P(B)=\frac{2}{3}

Now \bar{A}\cap B is represented as shaded areas in figure

Thus P(\bar{A}\cap B)=P(B)-P(A\cap B)

=\frac{2}{3}-\frac{1}{4}=\frac{5}{12}


Option 1)

5/12

This option is correct

Option 2)

3/8

This option is incorrect

Option 3)

5/8

This option is incorrect

Option 4)

1/4     

This option is incorrect

Posted by

Plabita

View full answer

JEE Main high-scoring chapters and topics

Study 40% syllabus and score up to 100% marks in JEE